LSAT and Law School Admissions Forum

Get expert LSAT preparation and law school admissions advice from PowerScore Test Preparation.

 Administrator
PowerScore Staff
  • PowerScore Staff
  • Posts: 8916
  • Joined: Feb 02, 2011
|
#36968
Complete Question Explanation

Resolve the Paradox. The correct answer choice is (D)

The stimulus presents what purports to be a discrepancy, and asks us to explain how both of the
following claims could be true:

1. A certain medication effectively lowers cholesterol.
2. Those using the medication have higher cholesterol levels than average.

It must be the case that those who are on this medication had their cholesterol reduced, while as a group
maintaining a level higher on average than that of the overall population.

Answer choice (A): The relative effectiveness of another medication is completely irrelevant to the
specific effects of this medication.

Answer choice (B): The effects of other medications do not explain the effects of this medication, so this
choice is incorrect.

Answer choice (C): What happens to people who do not take the drug offers no insight into what
happens to people who take the drug.

Answer choice (D): This is the correct answer choice, as it explains why those on the medication
could have derived benefits but stayed at a higher level than that of the overall population.

Answer choice (E): The average distribution of cholesterol in the population does not help to explain
why people receiving a cholesterol-lowering drug, nevertheless, have high cholesterol.

Get the most out of your LSAT Prep Plus subscription.

Analyze and track your performance with our Testing and Analytics Package.